Đến nội dung

Hình ảnh

chứng minh...


  • Please log in to reply
Chủ đề này có 9 trả lời

#1
diemdaotran

diemdaotran

    Trung sĩ

  • Thành viên
  • 101 Bài viết

chứng minh: $\frac{x^3+y^3+z^3}{3}\geq xyz+\frac{3}{4}\left | (x-y)(y-z)(z-x) \right |\forall x;y;z\geq 0$


                                                                                                                               $\sqrt{M}.\sqrt{F}=\sqrt{MF}$   


#2
nguyenquangtruonghktcute

nguyenquangtruonghktcute

    Hạ sĩ

  • Thành viên
  • 61 Bài viết

17903619_1833942450264419_72203187058320


Bài viết đã được chỉnh sửa nội dung bởi nguyenquangtruonghktcute: 15-04-2017 - 20:05


#3
NHoang1608

NHoang1608

    Sĩ quan

  • Thành viên
  • 375 Bài viết

cái đoạn này nè bạn:$(x+y+z)(x^{2}+y^{2}+z^{2}-xy-yz-zx)=\frac{1}{2}((x-y)+(y-z)+(z-x))((x-y)^{2}+(y-z)^{2}+(z-x)^{2})$


The greatest danger for most of us is not that our aim is too high and we miss it, but that it is too low and we reach it.

----- Michelangelo----


#4
nguyenquangtruonghktcute

nguyenquangtruonghktcute

    Hạ sĩ

  • Thành viên
  • 61 Bài viết

cái đoạn này nè bạn:$(x+y+z)(x^{2}+y^{2}+z^{2}-xy-yz-zx)=\frac{1}{2}((x-y)+(y-z)+(z-x))((x-y)^{2}+(y-z)^{2}+(z-x)^{2})$

bạn sửa lại đoạn đó được không



#5
victoranh

victoranh

    Trung sĩ

  • Thành viên
  • 102 Bài viết

hình như đến đó là sai à 


-----Đừng chọn sống an nhàn trong những năm tháng mà bạn "chịu khổ được"-----


#6
diemdaotran

diemdaotran

    Trung sĩ

  • Thành viên
  • 101 Bài viết

17795778_1833335043658493_70731468885479

ae kiểm tra hộ với, sao lại chứng minh ra nó >=9/2 nhỉ

sao ban ap dung đc bđt côsi cac hiệu đã dương đâu


                                                                                                                               $\sqrt{M}.\sqrt{F}=\sqrt{MF}$   


#7
nguyenquangtruonghktcute

nguyenquangtruonghktcute

    Hạ sĩ

  • Thành viên
  • 61 Bài viết

sao ban ap dung đc bđt côsi cac hiệu đã dương đâu

ừ, bài này sai rồi 



#8
Nguyenhuyen_AG

Nguyenhuyen_AG

    Trung úy

  • Thành viên nổi bật 2016
  • 945 Bài viết

Lời giải của bạn nguyenquangtruonghktcute này chỉ cần sửa lại như vầy

20175eb14c5c-ba05-47b7-9e38-a75089968547

Vì ta luôn có $x+y \geqslant |x-y|.$


Nguyen Van Huyen
Ho Chi Minh City University Of Transport

#9
Minhnksc

Minhnksc

    Sĩ quan

  • Điều hành viên OLYMPIC
  • 302 Bài viết

mình nghĩ là có kết quả mạnh hơn bài đã cho:

$\frac{x^3+y^3+z^3}{3}\geq xyz+\frac{9}{4}\left | x-y \right |\left | y-z \right |\left |z-x \right |$ với x;y;z dương

Hình như kết quả này đăng trên THTT số tháng 11 năm ngoái ở chuyên đề gì gì đấy thì phải.


Bài viết đã được chỉnh sửa nội dung bởi Minhnksc: 15-04-2017 - 19:54

Sống khỏe và sống tốt :D


#10
Nguyenhuyen_AG

Nguyenhuyen_AG

    Trung úy

  • Thành viên nổi bật 2016
  • 945 Bài viết

mình nghĩ là có kết quả mạnh hơn bài đã cho:

$\frac{x^3+y^3+z^3}{3}\geq xyz+\frac{9}{4}\left | x-y \right |\left | y-z \right |\left |z-x \right |$ với x;y;z dương

Hình như kết quả này đăng trên THTT số tháng 11 năm ngoái ở chuyên đề gì gì đấy thì phải.

 

Hằng số $k$ lớn nhất để bất đẳng thức

\[a^3+b^3+c^3-3abc \geqslant k|(a-b)(b-c)(c-a)|,\]

luôn đúng là $k=k_0 = \sqrt{9+6\sqrt3}.$


Bài viết đã được chỉnh sửa nội dung bởi Nguyenhuyen_AG: 16-04-2017 - 13:59

Nguyen Van Huyen
Ho Chi Minh City University Of Transport




1 người đang xem chủ đề

0 thành viên, 1 khách, 0 thành viên ẩn danh